Fórmula de Kubo para el efecto Hall cuántico

Estoy tratando de entender la fórmula de Kubo para la conductividad eléctrica en el contexto del efecto Hall cuántico.

Mi problema es que varios artículos, por ejemplo, el famoso artículo de TKNN (1982) , o una elaboración de Kohmoto (1984) , escriben las entradas diagonales del tensor de conductividad en la forma

σ X y ( ω 0 ) = i mi 2 mi a < mi F < mi b a | v X | b b | v y | a a | v y | b b | v X | a ( mi a mi b ) 2 .

Este es el límite estático. ω 0 y baja temperatura T 0 . La suma cubre todos los estados propios | a y | b del hamiltoniano de una sola partícula. mi F es la energía de Fermi. v X y v y son los operadores de velocidad de una sola partícula.

Sin embargo, estos documentos no derivan esta ecuación, lo cual es desafortunado porque la fórmula de Kubo generalmente no se presenta de esta forma. Encontré (y logré volver a derivar) la siguiente variación en su lugar

σ X y ( ω + i η ) = i mi 2 V ( ω + i η ) a , b F ( mi a ) ( a | v X | b b | v y | a ω + i η + mi a mi b + a | v y | b b | v X | a ω i η + mi a mi b ) .

Esta es la fórmula (13.37) de Ashcroft, Mermin , aunque en realidad no lo demuestran. F ( mi ) es la distribución de Fermi. Se da una buena derivación en Czycholl (alemán).

Ahora, mi pregunta es, obviamente

¿Cómo derivar la primera fórmula de la segunda?

Puedo ver que la primera ecuación surge como el término lineal al escribir la suma como una serie de potencias en ω , pero ¿por qué el término constante no diverge?

No estoy del todo seguro de esto, pero: creo que el problema puede ser que la conductividad de Hall se define como un componente antisimetrizado del tensor de conductividad, es decir, la cantidad a la que se aplica la primera fórmula en realidad puede ser σ X y σ y X . ¿Suena esto plausible?
No estoy seguro, pero una observación relacionada es que el tensor de conductividad probablemente debería ser antisimétrico en primer lugar.
Eso no me suena bien; debe haber conductividades ordinarias (no Hall) en la diagonal.
No tengo ni idea. ¿Podría dar un ejemplo de un material con conductividades diagonales? ¿O algún consejo general sobre estas cosas? Después de todo, las conductividades diagonales son extrañas porque la corriente fluye perpendicular al campo eléctrico aplicado.
¡Lo encontré! Se puede usar una ligera variación de un argumento de Czycholl para mostrar que el término divergente en realidad desaparece. Lo escribiré pronto.
¿Aplica la fórmula de kubo que derivó anteriormente a los cálculos numéricos junto con el método de enlace estricto?
Otra fuente para esto es el libro de texto de Chakraborty sobre el Efecto Hall Cuántico en el Apéndice B.

Respuestas (2)

De hecho, la primera fórmula se sigue de la segunda fórmula si dejamos ω 0 . Para ver eso, expande las fracciones como

1 ± ω + mi a mi b = 1 mi a mi b ( 1 ω mi a mi b ) + O ( ω 2 )

para obtener σ X y = σ 1 + σ 2 como la suma de un término potencialmente divergente

σ 1 = i mi 2 V ω a , b F ( mi a ) a | v X | b b | v y | a + a | v y | b b | v X | a mi a mi b

y un término que se parece a la primera fórmula

σ 2 = i mi 2 V a , b F ( mi a ) a | v X | b b | v y | a + a | v y | b b | v X | a ( mi a mi b ) 2 .


Para ver que el primer término desaparece en lugar de divergir, tenemos que usar la ecuación de movimiento de Heisenberg v X = d d t X = [ H 0 , X ] lo que da

a | v X | b = a | H 0 X X H 0 | b = ( mi a mi b ) a | X | b

y por lo tanto

a | v X | b b | v y | a + a | v y | b b | v X | a = ( mi a mi b ) ( a | X | b b | v y | a a | v y | b b | X | a ) .

Los factores ( mi b mi b ) cancelar y la suma restante más b se convierte en una suma sobre la identidad b | b b | = 1 . Así, llegamos a

σ 1 = i mi 2 V ω a , b F ( mi a ) ( a | X v y v y X | a ) = 0 .

desde el conmutador [ X , v y ] desaparece


Para ver que el segundo término es correcto, tenemos que acertar los índices de suma. Para hacer eso, tenemos que reorganizar la suma para obtener

σ 2 = i mi 2 V a , b ( F ( mi a ) F ( mi b ) ) a | v X | b b | v y | a ( mi a mi b ) 2 .

en el limite T 0 , la diferencia de las distribuciones de Fermi-Dirac F ( mi a ) F ( mi b ) será igual a

  • 1 si mi a < mi F < mi b
  • 1 si mi b < mi F < mi a
  • 0 de lo contrario

Usar esto y reorganizar la suma nuevamente da la fórmula de Kubo en la primera forma.

Hola, creo que tu prueba de la desaparición del primer término es incorrecta. Por ejemplo, si tenemos una superficie de Fermi, entonces el primer término aparentemente no es cero; es físicamente sensible que una conductividad tenga una divergencia de 1/omega en un metal.
@XuYang ¿Qué término cree que aparentemente no es cero? Tenga en cuenta que este cálculo se aplica solo a la conductividad transversal, no a la conductividad longitudinal, que de hecho puede divergir.
@GregGraviton Por ejemplo, el efecto Hall anómalo en un metal no es cero. Y, de hecho, puede ser capturado por la fórmula de Kubo, que resulta ser el primer término que anotaste.
Consulte la ecuación 1.12 en el siguiente artículo y la derivación posterior: journals.aps.org/prb/pdf/10.1103/PhysRevB.48.11705 En este artículo se muestra que el primer término solo desaparece para un aislante, no para un metal.
Por encima de la ecuación. (2.12) de las notas de Tong sobre el QHE, afirma que la desaparición del término divergente se puede ver a partir de la invariancia de calibre / conservación de la corriente. ¿Sabes cómo se mostraría eso?
@Dwagg Ni idea. De hecho, no lo creo: un argumento general basado en la invariancia del calibre probablemente también se aplicaría a la conductividad longitudinal. Pero se sabe que en un metal limpio y sin desorden, la conductividad longitudinal es en realidad infinita en el límite ω 0 . O el argumento que menciona contiene otro ingrediente que lo hace inaplicable a la conductividad longitudinal, o probablemente no sea válido.
No hay razón para el conmutador. [ X , v y ]   [ X , [ H , y ] ] para desaparecer. Por ejemplo, si tiene un término como pags 4 en el hamiltoniano. Contiene términos cruzados de la forma pags X 2 pags y 2 . no lo haría [ X , [ H , y ] ] ser distinto de cero en este caso. Su prueba parece funcionar solo para sistemas sin tales términos cruzados.
@ symandzik138 Vaya, ese es un buen punto. En un modelo de enlace estrecho, el hamiltoniano es una suma de exponenciales de la forma Exp ( i k X norte + i k y metro ) , y v X = H / k X , por lo que su argumento se mantiene. Necesito pensar en esto, la verdadera razón es probablemente que el sistema es aislante, es decir, la función de distribución F ( mi a ) contiene solo bandas completas.
He estado tratando de encontrar el argumento correcto en la literatura por un tiempo. Hasta ahora ningún éxito. Sospecho que, en general, la resistencia de Hall debe definirse como la parte antisimétrica de σ X y . Pero la prueba en el apéndice B de la revisión de QHE de Tapash Chakraborty parece estar bien a primera vista. Utiliza un punto de partida diferente. Así que es un poco difícil de comparar con esto.

Una buena derivación de la segunda fórmula se da en http://www.damtp.cam.ac.uk/user/tong/kintheory/four.pdf

Bienvenido a Physics.Stackexchange. Tenga en cuenta que para este sitio es mejor evitar las respuestas de solo enlace. Intente incluir la parte esencial del texto vinculado en su respuesta. El objetivo es hacer de SE una fuente de información y no solo una lista de enlaces.